Is Log(x) Continuous on the Interval (0, ∞)?

  • Thread starter Thread starter Yagoda
  • Start date Start date
  • Tags Tags
    Continuous
Click For Summary
The discussion centers on proving that the function f(x) = log(x) is continuous on the interval (0, ∞) by leveraging its continuity at x = 1 and the property log(xy) = log(x) + log(y). Participants explore using the epsilon-delta definition of continuity, suggesting that continuity at x = 1 can be extended to other points in the interval. One approach involves rewriting log(x) in terms of log(1) to facilitate the proof. The conversation highlights the challenge of applying the two given facts effectively to demonstrate continuity across the entire interval. Ultimately, the aim is to establish a valid proof that confirms the continuity of the logarithmic function.
Yagoda
Messages
45
Reaction score
0

Homework Statement


Prove that f(x)=\log x is continuous on (0, \infty) using that
(1) f is continuous at x=1 and
(2) \log(xy) = \log(x) + \log(y)



Homework Equations


The definition of continuity: for all \epsilon >0, there exists a \delta>0 such that if |x-x_0| < \delta then |f(x) - f(x_0)| < \epsilon.


The Attempt at a Solution


I think I've figured out how to do this using a more standard epsilon-delta proof, but it doesn't really make use of the two facts.
From what I can tell, it seems like you trying to be able to use the continuity at x=1 to "slide" the continuity down to 0 and up to infinity, but I'm not sure how to do this in a valid way. The only way I've managed to use fact 2 is rewrite things like \log x = \log(x \times 1) = \log(x)+\log(1) = \log(x), which hasn't gotten me very far.
 
Physics news on Phys.org
Try replacing x_0 with xy for some y.
 
Last edited:
  • Like
Likes 1 person
Look at ##f(x) - f(x_0)##, this becomes ##log(x) - log(x_0)##.
 
hint
log(x+h)-log(x)=log(1+h/x)-log(1)
 
Question: A clock's minute hand has length 4 and its hour hand has length 3. What is the distance between the tips at the moment when it is increasing most rapidly?(Putnam Exam Question) Answer: Making assumption that both the hands moves at constant angular velocities, the answer is ## \sqrt{7} .## But don't you think this assumption is somewhat doubtful and wrong?

Similar threads

Replies
22
Views
2K
  • · Replies 2 ·
Replies
2
Views
1K
  • · Replies 40 ·
2
Replies
40
Views
4K
Replies
7
Views
1K
  • · Replies 1 ·
Replies
1
Views
1K
  • · Replies 26 ·
Replies
26
Views
3K
  • · Replies 8 ·
Replies
8
Views
2K
Replies
6
Views
2K
  • · Replies 2 ·
Replies
2
Views
2K
  • · Replies 9 ·
Replies
9
Views
3K